Someone help me please!

Someone Help Me Please!

Answers

Answer 1

Answer:

y = 4x + 8

Step-by-step explanation:

1) Find slope:

y2-y1/x2-x1

12-8/1-0 = 4/1 = 4

Slope = 4

2) Plug in a point in point-slope form

y-y1=m(x-x1)

y-8=4(x-0) =

y-8=4x-0 -> add 8 to both sides

y=4x+8


Related Questions

What is the simplified expression for 6^4.6^3 over 6^5
6^1
6^2
6^3
6^7

Answers

Answer:

6^2

Step-by-step explanation:

Question:-To simplify the given expression

Expression:-[tex] \frac{ {6}^{4} . {6}^{3} }{ {6}^{5} } [/tex]Solution:-

[tex] = \frac{ {6}^{4} . {6}^{3} }{ {6}^{5} } [/tex]

[As we know, a^x . a^y = a^(x+y)]

[tex] = \frac{ {6}^{(4 + 3)} }{ {6}^{5} } [/tex]

[On Simplification]

[tex] = \frac{ {6}^{7} }{ {6}^{5} } [/tex]

[As we know, a^x / a^y = a^(x-y)]

[tex] = {6}^{(7 - 5)} [/tex]

[On Simplification]

[tex] = {6}^{2} (ans)[/tex]

ASAP HELP *Can a triangle have sides with the given lengths? Explain, showing all the work
9 cm, 11 cm, 15 cm
please Explain

Answers

Answer:

yes

Step-by-step explanation:

For the sides to form a triangle , then the sum of any 2 sides must be greater than the third side

9 + 11 = 20 > 15

9 + 15 = 24 > 11

11 + 15 = 24 > 9

The 3 sides given will form a triangle

Note that

The length of any two sides must be greater than the third side

So

9+11=16>1511+15=26>99+15=24>11

Yes it's a triangle

winch one is biger
4/9 0r 1/12

Answers

Answer:

4/9

Step-by-step explanation:

I did the butterfly method to see which one is the bigger one

Answer:

4/9

Step-by-step explanation:

23 base 10+1011 base 2=​

Answers

Answer:

10111

Step-by-step explanation:

10111

Convert 45 mi/hr to feet per minute

Answers

Answer:

3960

Step-by-step explanation:

multiply 45 by the amount of feet in a mile (5280)

take that answer and divide in by how many minutes are in an hour

A 2-column table with 6 rows. The first column is labeled x with entries negative 3, negative 2, negative 1, 0, 1, 2. The second column is labeled f of x with entries 50, 0, negative 6, negative 4, negative 6, 0. Use the table to complete the statements. The x-intercepts shown in the table are and. The y-intercept shown in the table is.

Answers

Answer:

Use the table to complete the statements.

The x-intercepts shown in the table are  

(–2, 0)

and  

(2, 0

.

The y-intercept shown in the table is  

(0, –4)

.

Step-by-step explanation:

i need help with this problem, but i don't want someone to just give me the answer. i need it to be taught step-by-step to me because for some reason i can't understand how everyone tells me to do it. i need to learn this as I've been on this one test for the past 2 weeks. I've been trying to find a tutor or something that will actually help me the way i need to be helped. so please, just give me a simple answer that will help me with the 3 other questions after it.

the lesson name is called solving real world systems of equations via elimination.
here is the question:

jared bought 7 cans of paint. a can of red paint costs 3.75. a can of black paint costs 2.75. jared spent 22.25 in all. how many cans of black (b) and how many cans of red (r) did he buy?

r+b=7

3.75 + 2.75 = 22.25

this is a lesson in 8th grade math.​

Answers

Answer:

Hallo, I will help you with the problem T^T

Step-by-step explanation:

3.75r + 2.75b = 22.25

3.75r = The amount a red can costs, MULTIPLIED BY the amount of red cans they bought!

2.75b = The amount a black can costs, MULTIPLIED BY the amount of black cans they bought!

So this means that they bought:

3 cans of red paint.

And 4 cans of black paint.

Now you may be thinking, how do you know?

If you multiply the COST 3.75, by 3 cans of red paint, you get 11.25

And then if you multiply the COST of the black paint (The cost for a can of black paint is 2.75) multiply that by 4, you get 11 :0

So add 11 + 11.25 and you get 22.25 :]

Comment if you have any questions, no matter how insignificant you think they may be, I will help you :]

Helpppppppppp asappppppppp 20 points

Answers

Here’s a small hint: “miles per hour”

Put the amount of miles over the amount of hours then calculate the amount from there! It’s either the 2nd option or the 4th one! Hope it’s helpful!

Answer:

i say its c

Step-by-step explanation:

What is the estimate height of 129.3

Answers

129 .................

PLEASE HELP!!
This is worth 20 points please help me!

Answers

If it’s worth 20 why you just giving 5 points boo

Challenge question? Thanks.

Answers

Answer:

x=15

Step-by-step explanation:

the basic property is: AB/AD=BC/CD, for more info see the attachment.

a) A triangular plot is being constructed having its base twice to its altitude. What will be the base and altitude of the plot if the cost of levelling it at the rate of ₹25 per square meter is ₹11,025.

b)If the same plot from above question is being reconstructed into a square field, having the same area, then find the perimeter of square


THE BEST ANSWER WILL BE MARKED BRAINLIEST PLEASE GIVE PROPER ANSWER
THANK U

Answers

The base and altitude of the triangular plot are [tex]42m[/tex] and [tex]21m[/tex] respectively.

The perimeter of the square plot having the same area as the triangular plot is [tex]84m[/tex]

a) Calculate the base and altitude of the triangular plot

The relationship between the Total cost of leveling the triangular plot, the cost per unit area, and the area of the triangular plot is

[tex]\text{unit cost}=\frac{\text{total cost}}{\text{area}}\\\\\text{area}=\frac{\text{total cost}}{\text{unit cost}}\\\\=\frac{\text{11025Rs.}\rupee}{25Rs./m^2}=441m^2[/tex]

The formula for calculating the area of a triangle, given the base and altitude is

[tex]\text{area of triangle}=\frac{1}{2}\text{base}\times\text{altitude}[/tex]

From the question

[tex]\text{base}=2\times\text{altitude}[/tex]

so

[tex]\text{area of triangle}=\frac{1}{2}\times 2\times\text{altitude}\times\text{altitude}\\\\\text{area of triangle}=\text{altitude}^2\\441=\text{altitude}^2\\\text{altitude}=\sqrt{441}=21m[/tex]

from this we can get the base

[tex]\text{base}=2\times\text{altitude}\\=2\times21m=42m[/tex]

b) Calculate the perimeter of a square plot having the same area

Since we found the area of the triangular plot to be [tex]441m^2[/tex], we will use this value for the area of the square plot, so that we can obtain the side

[tex]\text{area of square plot}=side^2\\441=side^2\\\sqrt{441}=side=21m[/tex]

Hence, the perimeter is equal to

[tex]perimeter=4\times side\\=4\times21=84m[/tex]

Learn more about area here: https://brainly.com/question/1447350

Find the value of x

Answers

Answer:

x = 8

Step-by-step explanation:

is an isosceles triangle, so 12 = x + 4

12 = x + 4

x = 12- 4

x = 8

----------------

check

12 = 8 + 4

12 = 12

the answer is good

what is the next term in the number pattern? 77, 64, 53, 44, 37

Answers

Answer:

32

...when you subtract five from 37

It is a pattern that goes:

-13

-11

-9

-7

and now -5 = 32

Solve the equations below. Give exact solutions only. Show your work.

a) 5x^2 - 7 = 72
b) x(x + 2) = 5
c) 3x^2 + 2x - 4 = 0

Please help!

Answers

Step-by-step explanation:

remember, the general formula to solve a quadratic equation ax² + bx + c = 0 is

x = (-b ± sqrt(b² - 4ac))/(2a)

a)

I think there is a typo, and this should be

5x² - 7x = 72

5x² -7x - 72 = 0

x = (7 ± sqrt(49 - -1440))/10 = (7 ± sqrt(1489))/10

x1 = (7 + sqrt(1489))/10

x2 = (7 - sqrt(1489))/10

b)

x(x + 2) = 5

x² + 2x - 5 = 0

x = (-2 ± sqrt(4 - -20))/2 = (-2 ± sqrt(24))/2 =

= (-2 ± sqrt(4×6))/2 = (-2 ± 2×sqrt(6))/2 =

= -1 ± sqrt(6)

x1 = -1 + sqrt(6)

x2 = -1 - sqrt(6)

c)

3x² + 2x - 4 = 0

x = (-2 ± sqrt(4 - -48))/6 = (-2 ± sqrt(52))/6 =

= (-2 ± sqrt(4×13))/6 = (-2 ± 2×sqrt(13))/6 =

= (-1 ± sqrt(13))/3

x1 = (-1 + sqrt(13))/3

x2 = (-1 - sqrt(13))/3

Some please help me I’ll give you brainlist answer

Answers

Answer:

Step-by-step explanation:

The area of the square is 3 units by 3 units. because the area of a square is side length squared.

side length = 3

side length^2 = 3 * 3 = 9

The area of a circle is much harder to reason out. You could start with the area of a square and use the diameter is 3 units. But the corners are rounded and you don't know off hand what the area of one of them is, let alone all 4. You are given a formula for circles area, but you have no real idea what the formula actually means. Does it take the corner area into account? At the beginning levels, we don't know.

Answer:

The area of the square is 9 aka [tex]3^{2}[/tex]

You can find it easily by just counting the number of squares inside the square, or by counting the number of squares at its length then multiplying it by its width; so 3 x 3 = 9

It wouldn't be as easy to find the area of the circle because you don't have the radius, nor diameter, so basically, no quantitative values could be used to at least guess.

Will mark brainly!! :)

Answers

Answer:

(2,1)

Step-by-step explanation:

The answer is (2,1)

Please help!!!!!!!!!!!!!

Answers

9 + (-3) = 6
Because based on the image you add 9 and subtract 3

Hope you find this helpful!
Brainliest is much appreciated!

Answer:

B and C

Step-by-step explanation:

look at comments

Describe the translation in j(x)=(2x)+3 as it relates to the graph of the parent function?

Answers

Answer:

Horizontal compression by a factor of 2 followed by a vertical shift by positive 3

Step-by-step explanation:

a*f(k(x-d))+c

k=2

c=3

Solve the inequality.


-7x - 4 > 17

Answers

Answer:x<-3

Step-by-step explanation:

Answer:

x < -3

Step-by-step explanation:

[tex]-7x-4>17\\\\-7x-4+4>17+4\\\\-7x>21\\\\\frac{-7x}{-7}>\frac{21}{-7}\\\\x<-3[/tex]

What two expressions are equivalent to 6.5 divided by 1.3?

Answers

Answer:

65/13

650/130

Step-by-step explanation:

6.5/1.3

times 10: 65/13

times 100: 650/130

f(x)=x^2-3, find f(-2),f(4),and f(0)
f(-2)=
f(4)=
f(0)=

Answers

Step-by-step explanation:

f(-2)= -2^2-3= -7

f(4)= 4^2-3= 13

f(0)=0^2-3= -3

The cost of an Uber is $8.00 plus $1.25 per
mile. Write a function, f(x), to determine the cost of an x-mile Uber ride.

Answers

Answer:

Cost = 8 + 1.25x

Step-by-step explanation:

PT. 4 OF MY HW! STILL UNIT RATE​

Answers

Answer: plant 1 grows at a faster rate

Step-by-step explanation:


[tex] \sf \huge{ question \hookleftarrow}[/tex]


If [tex] \alpha \: and \: \beta [/tex] are roots of a equation " ax² + by + c ", then find the value of the following in terms of a , b and c ~


[tex] \boxed{ \boxed{ \sf \sqrt{ \alpha} + \sqrt{ \beta } = \: ?}}[/tex]


Answers

[tex]\underline{\bf{Given \:equation:-}}[/tex]

[tex]\\ \sf{:}\dashrightarrow ax^2+by+c=0[/tex]

[tex]\sf Let\:roots\;of\:the\: equation\:be\:\alpha\:and\beta.[/tex]

[tex]\sf We\:know,[/tex]

[tex]\boxed{\sf sum\:of\:roots=\alpha+\beta=\dfrac{-b}{a}}[/tex]

[tex]\boxed{\sf Product\:of\:roots=\alpha\beta=\dfrac{c}{a}}[/tex]

[tex]\underline{\large{\bf Identities\:used:-}}[/tex]

[tex]\boxed{\sf (a+b)^2=a^2+2ab+b^2}[/tex]

[tex]\boxed{\sf (√a)^2=a}[/tex]

[tex]\boxed{\sf \sqrt{a}\sqrt{b}=\sqrt{ab}}[/tex]

[tex]\boxed{\sf \sqrt{\sqrt{a}}=a}[/tex]

[tex]\underline{\bf Final\: Solution:-}[/tex]

[tex]\\ \sf{:}\dashrightarrow \sqrt{\alpha}+\sqrt{\beta}[/tex]

[tex]\bull\sf Apply\: Squares[/tex]

[tex]\\ \sf{:}\dashrightarrow (\sqrt{\alpha}+\sqrt{\beta})^2= (\sqrt{\alpha})^2+2\sqrt{\alpha}\sqrt{\beta}+(\sqrt{\beta})^2[/tex]

[tex]\\ \sf{:}\dashrightarrow (\sqrt{\alpha}+\sqrt{\beta})^2 \alpha+\beta+2\sqrt{\alpha\beta}[/tex]

[tex]\bull\sf Put\:values[/tex]

[tex]\\ \sf{:}\dashrightarrow (\sqrt{\alpha}+\sqrt{\beta})^2=\dfrac{-b}{a}+2\sqrt{\dfrac{c}{a}}[/tex]

[tex]\\ \sf{:}\dashrightarrow \sqrt{\alpha}+\sqrt{\beta}=\sqrt{\dfrac{-b}{a}+2\sqrt{\dfrac{c}{a}}}[/tex]

[tex]\bull\sf Simplify[/tex]

[tex]\\ \sf{:}\dashrightarrow \underline{\boxed{\bf {\sqrt{\boldsymbol{\alpha}}+\sqrt{\boldsymbol{\beta}}=\sqrt{\dfrac{-b}{a}}+\sqrt{2}\dfrac{c}{a}}}}[/tex]

[tex]\underline{\bf More\: simplification:-}[/tex]

[tex]\\ \sf{:}\dashrightarrow \sqrt{\alpha}+\sqrt{\beta}=\dfrac{\sqrt{-b}}{\sqrt{a}}+\dfrac{c\sqrt{2}}{a}[/tex]

[tex]\\ \sf{:}\dashrightarrow \sqrt{\alpha}+\sqrt{\beta}=\dfrac{\sqrt{a}\sqrt{-b}+c\sqrt{2}}{a}[/tex]

[tex]\underline{\Large{\bf Simplified\: Answer:-}}[/tex]

[tex]\\ \sf{:}\dashrightarrow\underline{\boxed{\bf{ \sqrt{\boldsymbol{\alpha}}+\sqrt{\boldsymbol{\beta}}=\dfrac{\sqrt{-ab}+c\sqrt{2}}{a}}}}[/tex]

The value  [tex]\sqrt{\alpha } +\sqrt{\beta }[/tex] in terms of a, b and c is [tex]\sqrt{{(\frac{b}{a})^2 } +2\sqrt{\frac{c}{a} } } \\[/tex]

Roots of a quadratic equation

Given the quadratic equation ax² + bx + c, the sum and product  of the roots are expressed as:

[tex]\alpha +\beta =-\frac{b}{a} [/tex][tex]\alpha \beta =\frac{c}{a} [/tex]

Get the value of the radical expression [tex]\sqrt{\alpha } +\sqrt{\beta } [/tex]

Taking the square of the expression will give:

[tex](\sqrt{\alpha } +\sqrt{\beta } )^2=(\sqrt{\alpha } )^2+(\sqrt{\beta } )^2+2\sqrt{\alpha \beta} [/tex]

Take the square root of both sides:

[tex]\sqrt{(\sqrt{\alpha } +\sqrt{\beta } )^2} =\sqrt{(\sqrt{\alpha } )^2+(\sqrt{\beta } )^2+2\sqrt{\alpha \beta} } \\ \sqrt{\alpha } +\sqrt{\beta }=\sqrt{{(\alpha }+{\beta} )+2\sqrt{\alpha \beta} } \\[/tex]

Substitute the product and the sum values into the expression to have:

[tex]\sqrt{\alpha } +\sqrt{\beta }=\sqrt{{(-\frac{b}{a})^2 } +2\sqrt{\frac{c}{a} } } \\\sqrt{\alpha } +\sqrt{\beta }=\sqrt{{(\frac{b}{a})^2 } +2\sqrt{\frac{c}{a} } } \\[/tex]

Hence the value  [tex]\sqrt{\alpha } +\sqrt{\beta }[/tex] in terms of a, b and c is [tex]\sqrt{{(\frac{b}{a})^2 } +2\sqrt{\frac{c}{a} } } \\[/tex]

Learn more on the roots of equation here: https://brainly.com/question/25841119

Find the perimeter and area of the figure below.

Answers

Answer:

A: 312  P: 80

Step-by-step explanation:

Answer:

A=120, P=60

Step-by-step explanation:

A new DVD player has a purchase price of $430. With the extended warranty, the total cost is $512. What percent of the purchase price is the price of the warranty? a. 16% b. 19% c. 24% d. 29% Please select the best answer from the choices provided A B C D.

Answers

The percent of the purchase price that is the price of the warranty is: b. 19%.

Using this formula

Purchase price percent=Total cost-Purchase price/Purchase price

Where:

Total cost=$512

Purchase price=$430

Let plug in the formula

Purchase price percent=$512-$430/$430

Purchase price percent=$82/$430

Purchase price percent=0.19×100

Purchase price percent=19%

Inconclusion the percent of the purchase price that is the price of the warranty is: b. 19%.

Learn more about purchase price here:https://brainly.com/question/6076645

What are the domain and range of the function below? Domain[0, ∞). range (-∞, ∞). Domain[0, ∞). Range (-∞,4). Domain(0,4). Range (-∞, ∞). Domain(-∞,4]. Range[0, ∞).

HELP IMMEDIATELY PLEASE

Answers

Answer:

Domain[0, ∞). Range (-∞,4).

Step-by-step explanation:

domain is your lowest x (zero) to your highest x (infinity)

range is the lowest y (negative infinity) to your highest y (four)

Answer:

its b

Step-by-step explanation:

What is an included angle and an included side?

A: Included Angle: A 90 deg angle
Included Side: The longest
side

B: Included Angle: An angle between two angles
Included Side: A side between two sides

C: Included Angle: An angle between two sides.
Included Side: A side between two angles

Answers

Answer:

C

Step-by-step explanation:

Last week antione had a begging balance of $90 in his account he wrote 3 checks for $10 each from his checking account , withdrew another $20 and finally made a deposit of $100. Write an expression that shows his final account balance at the end of the week

Answers

Answer:

$140

Step-by-step explanation:

90-(3x10)-20+100= 140

He started with $90: (90)

Then he wrote 3 checks of $10 each: -(3x10)

Then he withdrew $20: -20

Finally, he made a deposit of $100: +100

Thus, our equation would be: 90-(3x10)-20+100=

(You might want to simplify this expression; giving you 140)

Other Questions
Use the six Spanish terms below to write about something in a traditional classroom. Make sure to use the correct number(s) for the nouns and the correct present tense form of the verb. Write your answer in Spanish.mochilacartelloslasthe verb estaren Which is the best estimate for the average rate of change for the quadratic function graph on the interval 0 A pair of dice is rolled. Determine the probability of a result with: i. one die showing a 4 and the other a 5? Martha fue a una tienda de dulces a granel y compr 150 gramos de caramelos a $10.29 por cada 100 gramos y 220 gramos de chispas a $21.29 por cada 100 gramos. Cunto dinero gast en total? Drawing Conclusions According to Hobbes, what do people mistakenly assume when they advocate that a division of powers is good for a commonwealth? Why are conservation and proper utilisation of natural resources essential for the sustainable development of the nation? Give reasons.(long question answer) Grass, trees, shrubs, worms, birds, and deer that live in a certain area is known as what? how does the florida constitution differ from the u.s. constitution and its amendments? Two forces that act on very small distances (smaller than you can see ) are Need help please. High school Geometry A truck moves 60 kilometers east from point A to point B. At point B, It turns back west and stops 15 kllometers away from point A. What are thetotal distance and total displacement of the truck? The area of a rectangular patio is 21 18 square meters. The width of the patio is 3 14 meters. What is the length? Enter your answer as a mixed number in simplest form.The length of the patio is meters. Which of the following statements best explains how multitasking works in the human mind? 1. How many small squares are in Step 10?Image below A. 10B. 11C. 90D. 110 Jana earns $25 per week for mowing the lawn and deposits the money into her savings account. The amount of money in Janas account can be represented by the equation y = 25x + 125, where x is the number of weeks she has been mowing the lawn. What does the y-intercept of the graph represent in this situation? A force of 5000n is applied outwardly to each end of a 5. 0-m long rod with a radius of 34. 0 cm and a young's modulus of 125 108 n/m2. The elongation of the rod is: phy101. I need help on question D And you will have to look at the answer from question C !I need answer ASAP PLZ! First one to answer will get a brainliest! And plz nooooo scam link! Put the following equation of a line into slope-intercept form, simplifying all fractions. 3x+6y=-42 Deondra needs to order some new supplies for the restaurant where she works. The restaurant needs at least 624 glasses. There are currently 344 glasses. If each set on sale contains 20 glasses, write and solve an inequality which can be used to determine ss, the number of sets of glasses Deondra could buy for the restaurant to have enough glasses Name one advantage and one disadvantage to changing career paths.